ดูหนึ่งข้อความ
  #59  
Old 27 มิถุนายน 2011, 08:13
Amankris's Avatar
Amankris Amankris ไม่อยู่ในระบบ
กระบี่ธรรมชาติ
 
วันที่สมัครสมาชิก: 13 มกราคม 2007
ข้อความ: 2,492
Amankris is on a distinguished road
Default

#39
แปลกๆนะครับ ตรง $32=112_3$

#47
ให้เหตุผลตัวเลือกที่ 3 แบบนั้นไม่ได้นะครับ

#48
คูณเลขผิดครับ

#54
$\theta=150^\circ$ ได้นะครับ

#55
โจทย์ให้หาจำนวนชุดคำตอบครับ (ในที่นี้คือจำนวนชุด $(x,n)$)

#58
เห็นด้วยครับ - -"

27 มิถุนายน 2011 08:48 : ข้อความนี้ถูกแก้ไขแล้ว 5 ครั้ง, ครั้งล่าสุดโดยคุณ Amankris
ตอบพร้อมอ้างอิงข้อความนี้